Jump to content

Can you move in space?


Recommended Posts

On 7/18/2019 at 6:11 AM, Mordred said:

So one wire pushes against the other but both are fixed in their distance correct ?

 That won't work it won't matter how strong the field is with that scenario.

I have stressed this ever since

Link to comment
Share on other sites

5 minutes ago, Mordred said:

I have stressed this ever since

You have stressed this but I haven’t.

You are still assuming the forces are continuous when they are not. 

Have you seen my animation? If this principal is wrong explain how Instead of saying its wrong for reasons I’m not stating 

Edited by MPMin
Link to comment
Share on other sites

Doesn't matter what you think. I know these laws apply in the scenario described. So does everyone else involved in this thread.

 The denial by attrition tactic doesn't change that 

 You can check any textbook in fact I recommend you do. It's high school physics. The third law is clear.

"For any force there is an equal and opposite force." Every physics text book will state this.

 You did not apply this law 

Edited by Mordred
Link to comment
Share on other sites

I did apply this law, the emp from wire A cancels out its own effect on wire A as the magnetic field emanates outward from the wire equally in all directions - no law broken here.

The magnetic field travelling out from wire A no longer effects wire A because its detached - no law broken here 

When the magnetic pulse from A arrives at wire B it causes momentum in wire B  because magnetic fields are known to move wires, that’s how electric motors work and no law has been broken here either.

Where has Newtons law been violated?

Link to comment
Share on other sites

30 minutes ago, dimreepr said:

I watched my toaster very closely this morning, imagine my disappointment when it failed to fly.

Imagine my disappointment at seeing more snide remarks instead of seeing a logical argument countering my last post!

Link to comment
Share on other sites

7 hours ago, MPMin said:

I did apply this law, the emp from wire A cancels out its own effect on wire A as the magnetic field emanates outward from the wire equally in all directions - no law broken here.

The magnetic field travelling out from wire A no longer effects wire A because its detached - no law broken here 

When the magnetic pulse from A arrives at wire B it causes momentum in wire B  because magnetic fields are known to move wires, that’s how electric motors work and no law has been broken here either.

Where has Newtons law been violated?

Part one is correct part two is incorrect. Part two is incorrect. Here is Lorentz force on two wires

Notice the direction of force on the two wires. Reverse one polarity and notice the direction. Both are either pointing toward each other or both away from each other.

http://hyperphysics.phy-astr.gsu.edu/hbase/magnetic/wirfor.html

See image here

You applied the Lorentz force equations. So this applies however your two wires are attached to the same structure.

An electric motor follows the right hand rule in an orientation between the magnet and current  to induce spin. Use the RH rule for that orientation.

Go ahead try 10 amps on one wire with zero amps on the other you will still attract outward on the two wires or inward. 

Here is a quote from that page...

"Once you have calculated the force on wire 2, of course the force on wire 1 must be exactly the same magnitude and in the opposite direction according to Newton's third law."

 

 

Edited by Mordred
Link to comment
Share on other sites

5 minutes ago, Mordred said:

See the directions of force on that link due to Lorentz force and think about number 2. Hint the part about the field no longer affecting A.

Are you suggesting that the moment you stop the current in wire A that the Magnetic field remains intact to wire A? 

 

Link to comment
Share on other sites

10 minutes ago, dimreepr said:

Just an observation

Yes of course, and of all the threads on this site, you just happen to post your meaningless observation in this particular thread...

Link to comment
Share on other sites

5 minutes ago, MPMin said:

Yes of course, and of all the threads on this site, you just happen to post your meaningless observation in this particular thread...

This thread reminds my of an idea, that got WAY more press than it deserved, in the late 70's/early 80's about powering a spacecraft using opposing gyroscopes.

Link to comment
Share on other sites

22 minutes ago, Bufofrog said:

I am going to go out on a limb here and say there is no amount of evidence or experimentation that will change MPMin's mind.  But you guys are giving it a valiant effort.

Despite all the angst i do actually appreciate everyone’s meaningful contribution. I still have not seen a counter argument that accurately takes the full scope of my design into account in their rebuttal. I still think most of you don’t fully understand my design, so while your rebuttals make sense to you, your rebuttals do not completely correlate to my design. 

I’d appreciate it even more if you all went out on a limb and really tried to understand my design instead of assuming I’m just trying to defy any Newton’s laws. If your Prima face impression of my design is just I’m trying to defy newtons law you’ll most likely not try to understand what I’m saying.

My design doesn’t defy Newton’s laws:

In the first phase, a current is pulse in Wire A, the emp from wire A cancels out its own effect on wire A as the magnetic field emanates outward from the wire equally in all directions - no law broken here.

The next phase is to stop the current in wire A producing a short emp between the wires (instead of a magnetic field that extends from wire A to wire B as is shown in all worked examples because they only consider continuous currents instead of pulsed currents). The emp emanates outward from wire A and leaves wire A behind in all direction. As the emp travelling away from wire A it no longer effects wire A because its detached, there is a gap between the emp and wire A - no law broken here 

the next phase is when the magnetic pulse from A arrives at wire B, wire B has a brief current pulsed through it to create an emp at wire B, this causes a force on wire B but not on wire A thus moving the entire craft to the left as per my animation. no law has been broken here either.

Most of the rebuttals revert back to my design defying newtons laws but i have not seen an argument that demonstrates this referencing my design accurately 

 

 

Edited by MPMin
Link to comment
Share on other sites

27 minutes ago, MPMin said:

If your Prima face impression of my design is just I’m trying to defy newtons law you’ll most likely not try to understand what I’m saying.

Then break out the maths and explain.

Link to comment
Share on other sites

5 minutes ago, MPMin said:

I still have not seen a counter argument that accurately takes the full scope of my design into account in their rebuttal.

You have. But you either don't understand them or you just reject them as not being relevant.

In summary: the force on the wires is not what move the craft. You can tell this, because the force on the wires is the same if you put everything in a metal box and yet, in that case, there would be no movement (as you admit). How can the wires "know" if the force on them is supposed to move the craft or not? They can't.

The only reason this could produce some thrust is because of the different amount of energy (momentum) radiated front and back. And that difference is caused by the presence of the wire (not by the force on it).

 

 

Link to comment
Share on other sites

12 hours ago, MPMin said:

No one said anything about trying random stuff and don’t try telling anyone here that before every innovation was tried the results were correctly anticipated every time.

The anticipated results should be based on the proper application of theory.  You seem to be suggesting that you would expect a result that contradicts theory. Conservation of momentum must hold, and no thought experiment or design can change that...to overturn it you need a real experiment...

...yet you don't seem to want to overturn it, just ignore it so your design can work...that is a bad expectation for your design.

Conservation of momentum tells you you have made an error. 

 

 

 

You don't need to fully understand your design to discount it...you are expecting a result that is not consistent with the physics you are using to design it.

For it to work the physics has to be wrong and your design based on that physics has to be wrong as well.

What are the odds of that?

Link to comment
Share on other sites

5 hours ago, MPMin said:

Are you suggesting that the moment you stop the current in wire A that the Magnetic field remains intact to wire A? 

 

No absolutely not. Try again it should be obvious once you identify the force lines in that link 

It is two wires applying Lorentz force and it's obvious which direction the two (not one) force lines point.

 The image in the link clearly shows them...

Edited by Mordred
Link to comment
Share on other sites

19 hours ago, MPMin said:

If you think this wont work show me the reference to where it has been documented that pulsing emp between fixed wires wont work.

I provided analysis based on references to relevant sources. It is not what I think, it is what Einstein and others thought; that electromagnetic radiation carries momentum and their thoughts are supported by evidence. Maybe a more simplified and extended explanation of that part of mainstream physics is required. 

Edited by Ghideon
Link to comment
Share on other sites

6 hours ago, MPMin said:

Despite all the angst i do actually appreciate everyone’s meaningful contribution. I still have not seen a counter argument that accurately takes the full scope of my design into account in their rebuttal. I still think most of you don’t fully understand my design, so while your rebuttals make sense to you, your rebuttals do not completely correlate to my design. 

I’d appreciate it even more if you all went out on a limb and really tried to understand my design instead of assuming I’m just trying to defy any Newton’s laws. If your Prima face impression of my design is just I’m trying to defy newtons law you’ll most likely not try to understand what I’m saying.

My design doesn’t defy Newton’s laws:

In the first phase, a current is pulse in Wire A, the emp from wire A cancels out its own effect on wire A as the magnetic field emanates outward from the wire equally in all directions - no law broken here.

The next phase is to stop the current in wire A producing a short emp between the wires (instead of a magnetic field that extends from wire A to wire B as is shown in all worked examples because they only consider continuous currents instead of pulsed currents). The emp emanates outward from wire A and leaves wire A behind in all direction. As the emp travelling away from wire A it no longer effects wire A because its detached, there is a gap between the emp and wire A - no law broken here 

the next phase is when the magnetic pulse from A arrives at wire B, wire B has a brief current pulsed through it to create an emp at wire B, this causes a force on wire B but not on wire A thus moving the entire craft to the left as per my animation. no law has been broken here either.

Most of the rebuttals revert back to my design defying newtons laws but i have not seen an argument that demonstrates this referencing my design accurately 

 

 

Yeesh the minute to pulse wire A to wire B wire B pulls  back on wire A. See the Blooming link if you refuse to believe me. 

It clearly shows two not one Lorentz force lines in the two wire scenario. 

YOU MUST ACCOUNT FOR BOTH.

Not just wire A simply examining WIRE A is not accounting for all force lines involved.....

Lol this is quite similar to the EM drive impossibility that you see on Google. They may be using microwaves but microwaves are EM  waves. (If you do Google make sure you find the link where they showed Earth's gravitational field interfering with the tests. Yes your idea is different than the drives so it has its own problem set.

Edited by Mordred
Link to comment
Share on other sites

On 7/19/2019 at 8:29 AM, MPMin said:
On 7/17/2019 at 3:04 AM, Strange said:

As action and reaction are equal and opposite, I suppose you can think of it the other way round. That would be like saying that a rocket goes because the burning fuel pushes on the front of the reaction chamber but not on the back where the exhaust is(*). Which is not wrong, but is just ... weird. So if you want to think that the craft moves because the EMP pushes on the wire, I guess you can. That doesn't stop it being hopelessly inefficient.)

 

 

12 hours ago, Strange said:

You have. But you either don't understand them or you just reject them as not being relevant.

In summary: the force on the wires is not what move the craft. You can tell this, because the force on the wires is the same if you put everything in a metal box and yet, in that case, there would be no movement (as you admit). How can the wires "know" if the force on them is supposed to move the craft or not? They can't.

The only reason this could produce some thrust is because of the different amount of energy (momentum) radiated front and back. And that difference is caused by the presence of the wire (not by the force on it).

 

Don’t take this as an attack but the reason I don’t accept your argument/s is because they are circular and contradictory and inconclusive, see above - how can you expect me to draw a conclusion from inconstant information?

And ‘i admit’ that if you put any propulsion system in a box it wont go anywhere, case in point, this point is inconclusive.

7 hours ago, Ghideon said:

provided analysis based on references to relevant sources. It is not what I think, it is what Einstein and others thought; that electromagnetic radiation carries momentum and their thoughts are supported by evidence. Maybe a more simplified and extended explanation of that part of mainstream physics is required. 

I too have provided references and shown my workings so i think the way forward is to break down my design into individual phases like the strokes of an engine to see where the problem is 

15 hours ago, Mordred said:

Go ahead try 10 amps on one wire with zero amps on the other you will still attract outward on the two wires or inward. 

Here is a quote from that page...

Are you saying that if one of the wires in parallel carries 10 amps and the other has no current that the force of attraction or repulsion (depending on direction of current) will be equal between the two wires?

6 hours ago, Mordred said:

It clearly shows two not one Lorentz force lines in the two wire scenario. 

 YOU MUST ACCOUNT FOR BOTH.

Are you referring to the diagram that is using a continuous currents to show the magnetic field lines and resulting force lines?

12 hours ago, Strange said:

The only reason this could produce some thrust is because of the different amount of energy (momentum) radiated front and back. And that difference is caused by the presence of the wire (not by the force on it).

 

With reference to my design, just considering the first cycle, are you saying the radiation from A is blocked by wire B thus the unblocked or unbalanced radiation from A escaping out the back is what’s pushing the craft to the left? While you think about that, does the radiation from B have any effect on that cycle or is it just A?

11 hours ago, J.C.MacSwell said:

You don't need to fully understand your design to discount it...you are expecting a result that is not consistent with the physics you are using to design it.

 For it to work the physics has to be wrong and your design based on that physics has to be wrong as well.

What are the odds of that?

If you don’t fully understand my design how can you possibly conclude its not consistent with Newton’s laws?

 

To break down my design in to phases, lets consider the first phase of creating the emp from wire A. If a very short pulse of current is pulsed in the wire A an emp should emerge. The purpose of pulsing the current is to detach the emp from wire A. Does anyone disagree that once the current stops in wire A, the emp detaches from wire A thus no longer having any connection or effect to wire A. This leaves wire A free to move inside the emp ring. Is there anything wrong with this concept?

 

Edited by MPMin
Link to comment
Share on other sites

40 minutes ago, MPMin said:

 

 

If you don’t fully understand my design how can you possibly conclude its not consistent with Newton’s laws?

I will give you an example:

If say, I tell you I have a system inside a black box, and I tell you I can get more energy out from it than I put in on a continuous basis, with no loss of energy to the system over time, you should be able to tell me that is impossible based on the known laws of physics.

You don't need to know how it is designed, or what is going on inside the box.

You could dismiss it out of hand, and no one here would dispute your claim that my design will not work.

Without some major change in the laws of physics it simply will not work.

Link to comment
Share on other sites

Guest
This topic is now closed to further replies.
×
×
  • Create New...

Important Information

We have placed cookies on your device to help make this website better. You can adjust your cookie settings, otherwise we'll assume you're okay to continue.